Serie Combinación de Primavera

Estoy un poco confundido con esta pregunta de mecánica de la escuela secundaria. Donde tiene dos resortes con diferentes constantes de resorte, digamos k 1 y k 2 , los unimos y del resorte resultante, un extremo está unido a la masa y el otro a un extremo fijo.

aquí

Ahora la ecuación de movimiento de la masa estará dada por

d 2 d t 2 ( X 1 + X 2 ) = ( k 1 X 1 + k 2 X 2 )
dónde X 1 y X 2 son la longitud de los resortes con los que aumentaron.

Ahora, ¿cómo hago para resolver esta ecuación? no me digas que encuentre la constante de resorte equivalente. porque en wikipedia presuponen que podemos escribir F = k mi q ( X 1 + X 2 ) , No sé cómo saben que puedes hacer esto en absoluto. Necesito una información adicional para poder reducir la ecuación diferencial a una variable. Por favor, ayúdame con esto.

Respuestas (3)

Encontrar el resorte equivalente y resolver la ecuación 1DOF es un atajo a un problema más complejo. Entonces, si desea conocer el enfoque más complejo, siga leyendo.

El problema es un caso especial del problema general con dos resortes y dos masas. El caso especial es cuando una de las masas es cero.

Entonces el problema general tiene masa metro 1 (el punto negro) al final de k 1 y metro 2 (la caja gris) al final de k 2 .

Figura 1

Las ecuaciones de movimiento en términos de las posiciones de las masas X 1 y X 2 son como sigue:

(1) metro 1 X ¨ 1 = k 1 X 1 + k 2 ( X 2 X 1 ) metro 2 X ¨ 2 = k 2 ( X 2 X 1 )

Tenga en cuenta los usos op X 1 y X 2 para extensiones de resorte, y estoy usando estas variables como posiciones de masa. La extensión del primer resorte es igual a X 1 , pero la extensión de los segundos resortes es igual a X 2 X 1 .

Cualquier solución de lo anterior es la superposición de las dos respuestas de frecuencia naturales del sistema. Y hay una forma estándar de resolver este problema utilizando valores propios y vectores propios y algo de álgebra lineal.

Pero en este caso metro 1 = 0 lo que hace que las ecuaciones anteriores sean iguales a

(2) 0 = k 1 X 1 + k 2 ( X 2 X 1 ) metro 2 X ¨ 2 = k 2 ( X 2 X 1 )

lo que las convierte en un sistema DAE (ecuaciones algebraicas diferenciales). Aquí resuelves la primera ecuación para X 1 y reemplázalo en la segunda ecuación.

(3) metro 2 X ¨ 2 = ( k 1 k 2 k 1 + k 2 ) k mi q X 2

Entonces, la solución es la misma que resolver una ecuación diferencial de 1DOF en términos de X 2 usando el resorte equivalente k mi q . Tenga en cuenta que para cada solución de X 2 El valor de X 1 se encuentra a partir de (2) con

(4) X 1 = k 2 k 1 + k 2 X 2


Apéndice I

Las dos frecuencias naturales del sistema general son

(5) ω 1 2 = ( k 1 + k 2 metro 1 + k 2 metro 2 ) ( k 1 + k 2 metro 1 + k 2 metro 2 ) 2 4 k 1 k 2 metro 1 metro 2 2 ω 2 2 = ( k 1 + k 2 metro 1 + k 2 metro 2 ) + ( k 1 + k 2 metro 1 + k 2 metro 2 ) 2 4 k 1 k 2 metro 1 metro 2 2

Para el caso cuando metro 1 = 0 entonces lo anterior se convierte en

(6) ω 1 2 = ± ω 2 2 = k 1 k 2 k 1 + k 2 metro 2

Apéndice II

Las ecuaciones de movimiento en términos de extensiones de resorte. X 1 y X 2 son

(7) metro 1 X ¨ 1 = k 1 X 1 k 2 X 2 metro 1 X ¨ 1 + metro 2 X ¨ 2 = k 2 X 2

con la solución para metro 1 = 0 como

X 1 = k 2 k 1 X 2
y

X ¨ 1 + X ¨ 2 = k 2 metro 2 X 2

pero desde X 1 es una función de X 2 sólo entonces X ¨ 2 = k 2 k 1 X ¨ 1 y la ecuación 1DOF se convierte en

( 1 + k 2 k 1 ) X ¨ 2 = k 2 metro 2 X 2

que se vuelve equivalente a (3) cuando se resuelve para X ¨ 2 .

Parece que algo podría estar mal aquí. Parece que tienes la fuerza que actúa sobre la masa igual a k mi q X 2 ; pero no debería ser k mi q ( X 2 + X 1 ) ? Además, de las reglas sobre resortes en serie veo que X 1 = k 2 k 1 X 2 ; mientras que el tuyo también se ve diferente.
Lo siento mi X 1 y X 2 son la ubicación absoluta de los extremos de los resortes. No las extensiones relativas de los resortes.
OP usado X 1 y X 2 como la longitud de los resortes respectivos, por lo que podría ser muy confuso.
Veo eso, pero es común en estos problemas usar posiciones de nodo, así que solo estoy siguiendo procedimientos estándar. Edité la figura y el texto para reflejar eso.
Sigo pensando que es probable que cambiar las variables declaradas explícitamente de OP genere más confusión.
@JMac El uso de posiciones produce una ecuación de 1DOF en términos de X 2 . Pero el uso de extensiones de resorte produce la ecuación en términos de X 1 + X 2 Esto provoca la confusión, ya que parece ser una ecuación 2DOF. Pero X 1 = k 2 k 1 X 2 estaría implícito. La solución es mucho más simple con posiciones de masa que con extensiones de resorte.
@JMac: agregué el Apéndice II con las ecuaciones en términos de extensiones y no es bonito.

Como de costumbre, necesitas 2 ecuaciones para resolver un problema que contiene 2 variables. Comenzaría tratando de encontrar una relación entre X 1 y X 2 . Debería haber uno, ¿no? Si tira de un extremo de los resortes, la articulación debe estabilizarse en una posición proporcional a k 1 y k 2 .

Una pista es que la unión entre los resortes también sigue la tercera ley de Newton;)

Entendí el punto, gracias
Pero en este caso hay una masa cero unida al final del primer resorte, por lo que encontrarás una singularidad en las dos ecuaciones. La única forma de resolver esto con una ecuación y el resorte equivalente.

En wikipedia presuponen que podemos escribir F = k mi q ( X 1 + X 2 ) , No sé cómo saben que puedes hacer esto en absoluto.

Tiene bastante sentido para mí. Estás buscando el comportamiento de la masa al final. La masa se mueve debido a las fuerzas que actúan sobre ella desde los resortes.

Lo que pasa con la constante de resorte equivalente es que nos permite determinar la fuerza del sistema de resorte al simplificar múltiples resortes en un solo resorte que produce la misma fuerza para el mismo desplazamiento de la masa, por lo que al resolver el comportamiento del sistema equivalente , también obtenemos el comportamiento del sistema que desea, porque todo lo que afecta a la masa es la fuerza del resorte, y la fuerza del resorte se puede determinar por completo usando constantes de resorte equivalentes.

Intentaré mostrar cómo funciona para los resortes de serie en general.

Queremos llevarlo a un resorte equivalente donde:

F metro = k mi q ( X 1 + X 2 )

(Estoy llamando F metro la fuerza que actúa sobre la masa) Para resortes en serie, si los resortes no tienen masa, todos deben tener la misma fuerza actuando sobre ellos. Debido a esto, sabemos:

F metro = F 1 = F 2

Dónde F 1 es la fuerza sobre el resorte uno y F 2 es la fuerza sobre el resorte 2. Usando F 1 = X 1 k 1 y F 2 = X 2 k 2 y reorganizando para X 1 y X 2 obtenemos:

X 1 = F 1 k 1   X 2 = F 2 k 2

podemos sustituir X 1 y X 2 en la ecuación de la constante de resorte equivalente. Además, escribiré F metro , F 1 y F 2 tan solo F ahora ya sabemos que son todos equivalentes. Después de la sustitución obtenemos:

F = k mi q ( F k 1 F k 2 )

Deshacerse del doble negativo:

F = k mi q ( F k 1 + F k 2 )

Ahora si traemos F a la derecha, y k mi q a la izquierda, veremos la forma más familiar:

1 k mi q = ( 1 k 1 + 1 k 2 )

Entonces, puede ver en la derivación que usar la constante de resorte equivalente en realidad da la misma fuerza de salida que si sumara los desplazamientos de cada resorte por separado, por lo que solo necesita usar el desplazamiento equivalente total ( X 1 + X 2 ) como una variable junto con la única constante de resorte equivalente, en lugar de tratar con ambas por separado para obtener exactamente el mismo resultado en la masa.